LSAT and Law School Admissions Forum

Get expert LSAT preparation and law school admissions advice from PowerScore Test Preparation.

 Administrator
PowerScore Staff
  • PowerScore Staff
  • Posts: 8927
  • Joined: Feb 02, 2011
|
#25010
Complete Question Explanation

Justify the Conclusion. The correct answer choice is (A)

The economist states that since it would have cost nothing for Checker’s to accept competitor Marty’s coupons, their motivation for refusal to accept the coupons was to hurt Marty’s Pizza. The question stem asks us for what would allow the conclusion to be drawn, or what is sufficient for the conclusion that Checker’s motivation was to spite Marty’s. To prephrase, we want to think about the gap between the premises and the conclusion. We know that there was no cost to Checker’s to accept Marty’s coupons. But just because there was no cost, we cannot automatically assume that the motivation for denying the coupon was to hurt Marty’s. There may have been non-cost related reasons Checkers would deny the coupon. Therefore, the correct answer choice needs to eliminate the non-cost related possibilities.

Answer choice (A): This is the correct answer choice. According to answer choice (A), any company that refuses to accept coupons issued by a competitor is motivated solely by the desire to hurt that competitor. Since Checker’s would fall into the category of stores that refuse to accept competitor’s coupons, this answer choice would justify the conclusion that Checker’s motive in refusing to accept the coupons was simply to hurt Marty’s. Since Checker’s would fall into the category of stores that refuse to accept competitor’s coupons, this answer choice would justify the conclusion that they were acting in order to hurt Marty’s.

Answer choice (B): This is the equivalent of a Mistaken Reversal. The stimulus uses the fact that Checker’s refuses to accept the coupon as a premise, and draws the conclusion that they must be motivated by a desire to harm Marty’s. The correct answer choice must be also structured to lead to that conclusion. This answer choice leads to the conclusion that Checker’s would refuse to accept the coupon, which is not the correct conclusion.

Answer choice (C): This answer choice does not do enough to move from the premise to the conclusion. While it helps the conclusion, it does not provide enough information to show that the conclusion would have to be drawn. The answer choice is limited to some stores that refuse to accept the coupon, but in order for the argument in the stimulus to be valid, we need to know that all stores like Checker’s would refuse to accept the coupon just to hurt the competitor.

Answer choice (D): This answer choice provides a possible reason why Checker’s may not want to accept Marty’s coupons. However, it does not address the critical missing link in the stimulus.

Answer choice (E): In a Justify the Conclusion question, it is fine to use outside information in the answer choices. In fact, the correct answer choice will need to add some information to allow the conclusion to be drawn. However, this answer choice did not provide information that directly connects the premise and the conclusion.
 isoifer
  • Posts: 13
  • Joined: Jun 25, 2018
|
#47618
I chose C because for some reason I thought A would be too extreme. For assumption questions, is it safer to go with the more extreme answers at times since it removes all other possibilities? I thought that I was being safe by choosing C because it narrows down to Marty's (and potentially other stores) seeking to hurt competitors. I don't see how C doesn't enable the professor's conclusion to be drawn.
 Adam Tyson
PowerScore Staff
  • PowerScore Staff
  • Posts: 5153
  • Joined: Apr 14, 2011
|
#48148
This isn't an Assumption question, isoifer, but a Justify the Conclusion question (although some folks do refer to it as a Sufficient Assumption question, meaning the correct answer is sufficient to prove the conclusion is necessary). We typically don't like extreme language on Assumptions, but we LOVE them on Justify questions! After all, we are trying to prove the conclusion beyond any doubt, and that's a pretty extreme standard. Answer C tells us nothing about whether Checkers did what they did only to hurt Marty's. What does it matter that someone has done it for that reason, if that someone wasn't Checkers?

"If assumed" tells you that this is NOT an assumption question, because there is no "if" about assumptions. They are certain - the author MUST believe them. "Enables the conclusion to be properly drawn" = that's the language of a Justify question, because it means we want the conclusion to be absolutely and inescapably true. That's why the extreme language of A is so great, and exactly what we need.

Be careful in identifying question types so you know which strategy to apply! That's an easy mistake to make, and not one that you want to repeat.
 ericau02
  • Posts: 73
  • Joined: Feb 19, 2019
|
#64037
Hi I chose B because I really at first believed that A was to easy for it to be the correct answer, maybe I am looking the stimulus wrong but it seemed like A was a rebuttal of the conclusion.

I know that D is incorrect but I am a bit confused because (D) shows that checkers does not want to help showing its motives.

So I guess what I am asking is what makes A the correct answer choice in comparison to D. It seems as if they are both stating the same thing in different statements.
 ericau02
  • Posts: 73
  • Joined: Feb 19, 2019
|
#64038
Another questions I have is for ac (C) why do we need to know that ALL stores like checkers need to have that motive. I am having difficulty understanding the logic behind that switch of "any company" and "at least one" thanks
 Brook Miscoski
PowerScore Staff
  • PowerScore Staff
  • Posts: 418
  • Joined: Sep 13, 2018
|
#64078
Erica,

I suggest reading the initial explanation by the Administrator.

Your goal is to prove that Checkers wanted to hurt Marty's. This is a justify question, so you want 100% proof.

(B) is a Mistaken Reversal. Alternatively, it starts at the desire to cause harm--but that's what you need to prove, so the starting point and ending point are wrong.

(C) just says what one company, maybe others have done. You don't know whether Checkers is one of those companies. It isn't anywhere close to 100% proof about Checker's motives. That's why the "at least one" makes this choice wrong.

(D) is wrong because a desire not to help your competitor is very different from a desire to harm your competitor, so D undermines the stimulus by suggesting an alternative reason why Checkers didn't take the coupons.

I think that it's appropriate to be cautious on the LSAT. At the same time, you need to develop confidence in your comprehension and responses. It is early in your preparation, so I understand the impulse to make decisions and guesses based on thoughts and feelings that aren't on the page. I encourage you to drill down and keep yourself inside of the page--the stimulus and answer choices--for long enough to feel decisive. It won't always be possible, and you won't always be right, but you will learn more and will develop confidence.
 ericau02
  • Posts: 73
  • Joined: Feb 19, 2019
|
#64106
Great! Thanks Brooke your explanation helped much more and I am able to see why the ac's were wrong!!!
 falconbridge
  • Posts: 14
  • Joined: May 05, 2020
|
#80658
I didn't choose A for this question because the stimulus states "Accepting them would have cost Checkers nothing AND would have satisfied those of its potential customers who had purchased the coupon books."

Yet, answer choice A states "Any company that refuses to accept coupons issued by a competitor when doing so would satisfy some of the company’s potential customers is motivated solely by the desire to hurt that competitor."

I don't understand why this is correct. This answer states that if a company doesn't accept coupons when doing so would satisfy their customers then their motivation must be to spite their competitors; however, it leaves open the possibility that they didn't accept the coupons because it might have cost them money cost them money.

I'm not explaining this well, but in essence I see it as: the stimulus states a company's motivation is C when A and B are present, yet the answer choice states a company's motivation is C when only B is present.

Any help?
 Rachael Wilkenfeld
PowerScore Staff
  • PowerScore Staff
  • Posts: 1358
  • Joined: Dec 15, 2011
|
#80784
Hi Falcon,

It looks like you are confusing the conclusions and support for the conclusion.

The conclusion in this argument is that Checker's motive was to hurt Marty's. That's it.

The reasons given include that it would cost them nothing to accept the coupons, and would satisfy long term customers.

We need an answer choice that supports that conclusion--the only motivation was to hurt Marty's. That's what answer choice (A) gives us---any company that refuses coupons that could satisfy some customers (here, longer term customers) must be motivated by trying to hurt the competitor.

Make sure you focus hard on the conclusion---the specific conclusion. The conclusion may not be a full sentence, but only a phrase of a sentence. To find it, make sure that you are looking for a part of the argument that does not support any other part. If it is providing support to another part of the argument, it's not your main conclusion. It's a premise.

Hope that helps!

Get the most out of your LSAT Prep Plus subscription.

Analyze and track your performance with our Testing and Analytics Package.